RegistrierenRegistrieren   LoginLogin   FAQFAQ    SuchenSuchen   
Sphärische Bahnen
 
Neue Frage »
Antworten »
    Foren-Übersicht -> Mechanik
Autor Nachricht
Thomas1989
Gast





Beitrag Thomas1989 Verfasst am: 10. Nov 2010 16:52    Titel: Sphärische Bahnen Antworten mit Zitat

Meine Frage:
Hallo ich habe hier eine sehr doofe Frage in theoretischer Physik, bei der mir irgendwie der Ansatz fehlt:
Der Betrag eines Vektors sei zeitlich konstant. Zeigen Sie, dass die Vektoren und senkrecht aufeinander stehen.

Meine Ideen:
Wenn ich jetzt einen zeitlich konstanten Vektor ableite, dann fliegt das t doch bei der Ableitung raus, oder?
Aber dann habe ich doch die Vektoren a und a und diese sind doch dann nicht rechtwinklig.
DrStupid



Anmeldungsdatum: 07.10.2009
Beiträge: 5044

Beitrag DrStupid Verfasst am: 10. Nov 2010 17:18    Titel: Re: Sphärische Bahnen Antworten mit Zitat

Thomas1989 hat Folgendes geschrieben:
Meine Frage:
Hallo ich habe hier eine sehr doofe Frage in theoretischer Physik, bei der mir irgendwie der Ansatz fehlt:
Der Betrag eines Vektors sei zeitlich konstant. Zeigen Sie, dass die Vektoren und senkrecht aufeinander stehen.

Meine Ideen:
Wenn ich jetzt einen zeitlich konstanten Vektor ableite, dann fliegt das t doch bei der Ableitung raus, oder?


Da würde nicht nur das t rausfliegen. Die Ableitung einer Konstanten ist Null. Aber hier soll ja nicht der Vektor, sondern nur sein Betrag konstant sein.
thomas1989
Gast





Beitrag thomas1989 Verfasst am: 10. Nov 2010 17:44    Titel: Antworten mit Zitat

Das heißt jetzt im klartext. Also das hilft mir noch nicht ganz weiter.
mayap



Anmeldungsdatum: 15.12.2009
Beiträge: 301

Beitrag mayap Verfasst am: 10. Nov 2010 18:58    Titel: Antworten mit Zitat

Na, fang doch mal von vorne an, als Hilfe mal zwei Fragen:

- Was heisst denn, dass zwei Vektoren Senkrecht aufeinander stehen?
- Was heisst, dass der Betrag des Vektors zeitlich konstant ist?

Schreib das mal auf und vergleiche.
thomas1989
Gast





Beitrag thomas1989 Verfasst am: 10. Nov 2010 19:27    Titel: Antworten mit Zitat

Na also wenn 2 Vektoren aufeinander stehen ist ihr Skalarprodukt 0. Das wäre das erste was mir dazu einfällt. Das wäre auch Grundlage meines Beweises.
Das mit dem Betrag des Vektors ist zeitlich konstant kann ich mir iirgendwie nicht vorstellen.
DrStupid



Anmeldungsdatum: 07.10.2009
Beiträge: 5044

Beitrag DrStupid Verfasst am: 10. Nov 2010 21:30    Titel: Antworten mit Zitat

thomas1989 hat Folgendes geschrieben:
Das mit dem Betrag des Vektors ist zeitlich konstant kann ich mir iirgendwie nicht vorstellen.


Nimm einen Bleistift und stell Dir vor, es wäre ein Vektor. Solange Du den nur im Raum herumdrehst, bleibt sein Betrag konstant. Der ändert sich nur beim Anspitzen.
thomas1989
Gast





Beitrag thomas1989 Verfasst am: 10. Nov 2010 21:41    Titel: Antworten mit Zitat

Also ich glaube scheinbar liegt mein Problem jetzt bei der Ableitung von Vektoren. Gibts da ne allgemeine Form? Also wie man das generell macht. Werde aus meinen google Ergebnissen nicht ganz schlau.
mayap



Anmeldungsdatum: 15.12.2009
Beiträge: 301

Beitrag mayap Verfasst am: 11. Nov 2010 04:06    Titel: Antworten mit Zitat

Das weisst du wahrscheinlich schon, nur is die das wahrscheinlich nicht so bewusst smile

Eigentlich ganz einfach, wie man die ableitung eines vektors bildet:

Du weisst ja, dass die Ableitung des Ortes die geschwindigkeit ist:


gleichzeitig weisst du, dass der Geschwindigkeitsvektor aus den Geschwindigkeiten in jede Richtung gebildet wird, was die zeitableitung der jeweiligen Richtungen ist:



damit weisst du eigentlich, dass die Ableitung eines vektors einfach gebildet wird, indem die Komponenten abgeleitet werden und daraus ein vektor gebaut wird:



Dass das Skalarprodukt null ist, is hier schonmal der richtige Weg. Einfach mal aufschreiben und aus-x-en.

Parallel dazu mal die andere Bedingung aus-x-en und du solltest sehen, dass das recht gleich ist smile Dabei hilft vielleicht, dass die zeitliche Änderung des Betrags einfach:



ist. Den Betrag einfach einsetzen.

PS: nicht vergessen, dass die x, y, z da oben immer x(t), y(t), z(t) sind.
Neue Frage »
Antworten »
    Foren-Übersicht -> Mechanik